LSAT and Law School Admissions Forum

Get expert LSAT preparation and law school admissions advice from PowerScore Test Preparation.

 Administrator
PowerScore Staff
  • PowerScore Staff
  • Posts: 8916
  • Joined: Feb 02, 2011
|
#43377
Please post your questions below! Thank you!
 Khodi7531
  • Posts: 116
  • Joined: Mar 14, 2018
|
#45892
Although I clearly understand that this was a corr/cause issue, and detected it, I thought the issue it was asking us to point out had to do with the conclusions jump to "peoples health".


Lung disease is one aspect of "peoples health". It didn't say "overall health" which I assume would be more problematic but I was stuck between C and D and went with D. I was trying to think whether the author does assume doesn't have lung disease is in good health and during the timed test, I for some reason thought I saw some signs of it and wanted an answer that addressed "health" more.

Now without time I see that the author doesn't really assume this - right?

Any thoughts on how to approach this next time? Usually when we see a jump in language, i've gotten used to attacking that jump. I just don't think I would have been confident enough to say it's only corr/cause issue since there is "more" to it.


Can anyone help me out?



Also....is "beneficial effect" have any significance to this? Is that what pulls the issue to be more toward corr/cause and not the jump in language? How do I know which to go with next time...
 James Finch
PowerScore Staff
  • PowerScore Staff
  • Posts: 943
  • Joined: Sep 06, 2017
|
#45899
Hi Khodi,

You're not wrong to see that this argument has both an issue of assuming causation based only on correlation, as well as a part-to-whole issue. Lung health is certainly part of overall health, but only a part; one can't assume that just because aerobic exercise promotes lung health, it doesn't have equally or even worse effects on other areas of health (like knees/back/heart etc). Certainly the conclusion could be true, but the stimulus is far from proving it.

So an answer choice that reflects either the part-to-whole issue or the correlation/causation issue would be correct for this flaw question. In this case, only (C) reflects either flaw (the correlation/causation one), while (D) describes an assumption that the stimulus doesn't actually make, making it incorrect on factual grounds alone.

Hope this helps!
 Khodi7531
  • Posts: 116
  • Joined: Mar 14, 2018
|
#45919
Yeah I was just confused that the option wasn't given, but I do 100% see the cause/corr issue. Just didn't think they would test that question specifically on this since they usually make these questions with the jump in reasoning to address the jump and not the more simple underlying issue.


I just thought D said what I was thinking although it wasn't perfect. So that was also part of the issue for me.

For future reference, if I liked C and new that the issue that it addresses did in-fact exist, but D was using the words that I was looking for...how can I differentiate them or get myself comfortable enough to say, "this question is giving me a trap answer that makes me think it's addressing the overall health when it isn't...while C actually does address part of the issue"

I slowly read D because I knew it wasn't perfect and had a weird feeling about it - usually my gut saying i'm not really 100 percent with this and maybe it's cause i don't full get what it's saying but it has some of the words. Is that enough for me to get rid of D?
 nrpandolfo
  • Posts: 33
  • Joined: Feb 04, 2018
|
#46324
Hi,

Can you explain why answer choice C is correct instead of D? I thought the flaw was that the argument was leaping from having less risk of lung disease to having good health in general?

Thanks,
 Alex Bodaken
PowerScore Staff
  • PowerScore Staff
  • Posts: 136
  • Joined: Feb 21, 2018
|
#46563
nrpandolfo,

Thanks for your question! The issue with answer choice (D) is that it misdescribes what the author does...she doesn't presume that people are in good health, just that lowering the risk of lung disease has a beneficial effect on people's health. That's an important distinction - and it means that the author does not presume what answer choice (D) says she does. By contrast, she does make the error described in answer choice (C) by assuming causality between two events (increased rates of exercise and lowered rates of lung disease) when only correlation can be proven. That makes (C) the correct answer choice.

Hope that helps!
Alex
 mguitard
  • Posts: 9
  • Joined: Jul 16, 2020
|
#77860
Hello,

Like the others above I am confused how we chose C over D. However, I think I understood why D is incorrect, but still don't fully see how C is correct.

Would D be incorrect because the answer says, "anyone who does not have lung cancer is in good health, meaning that the answer would assume, basically, that everyone who doesn't have lung cancer is "in good health," but the argument never addresses others, just those who had lung disease?

As for my question with C, is the author assuming the correlation when he says "has a significant beneficial effect..."?

Thanks so much!
 Frank Peter
PowerScore Staff
  • PowerScore Staff
  • Posts: 99
  • Joined: May 14, 2020
|
#77935
Hi mguitard,

It sounds like you've got a pretty good understanding of what's happening here.

(D) is incorrect because it misstates what is happening in the argument. The author states that aerobic exercise benefits people's health. The author never states that anyone that doesn't have lung disease is in good health.

(C) identifies a pretty fundamental problem with this argument. There is a correlation between two factors (engaging in exercise and a low risk of lung disease), but it is never established how one might lead to the other. It invites challenges to the argument because one could easily identify potential alternate causes (i.e. someone who exercises also lives a healthy lifestyle generally).
User avatar
 AspenHerman
  • Posts: 61
  • Joined: Apr 03, 2021
|
#89249
Hi my powerscore friends,

After reading through the discussion on this question, I still have a question. Is D incorrect because it explicitly states "good health" as opposed to what the stimulus says that exercise has a "significant beneficial effect to people's health"? That last statement doesn't necessary make their health good, just better, which is why D is incorrect.

Aspen
 evank28
  • Posts: 2
  • Joined: Jul 26, 2021
|
#89262
Hi Aspen,

Yes, "good health" is not at all stated in the passage, so this makes D clearly wrong. Additionally, the stimulus never actually makes a presumption about "anyone who does not have long disease" it just extrapolates from a correlation; so it is incorrect to assume that the stimulus presumes something when it does not.

Get the most out of your LSAT Prep Plus subscription.

Analyze and track your performance with our Testing and Analytics Package.